LSAT and Law School Admissions Forum

Get expert LSAT preparation and law school admissions advice from PowerScore Test Preparation.

 Administrator
PowerScore Staff
  • PowerScore Staff
  • Posts: 8919
  • Joined: Feb 02, 2011
|
#35435
Complete Question Explanation

(See the complete passage discussion here: lsat/viewtopic.php?t=14270)

SR, Weaken. The correct answer choice is (A)

This question requires you to find the answer choice that would weaken the author’s argument in the
last two paragraphs, where the author asserts that the notion of clinical equipoise should be adopted,
in part because the absence of consensus among experts makes clinical equipoise possible.

Answer choice (A): This is the correct answer choice. The author’s argument is that lack of
consensus makes clinical equipoise practicable, because participants can have a preference but
recognize that experts disagree. If, as this choice provides, most comparative clinical trials seek to
confirm the preferability of a treatment that is considered the best by a consensus of doctors, the
author’s point no longer holds.

Answer choice (B): This choice is irrelevant to the argument presented by the author, so this choice
should be ruled out of contention.

Answer choice (C): The argument presented in the last two paragraphs of the passage deals neither
with the number of comparative trials nor the level of ethical oversight, so this is not the correct
response to this Weaken question.

Answer choice (D): The author’s argument has nothing to do with which of these two groups has a
stronger preference for a standard of theoretical equipoise.

Answer choice (E): This choice provides that researchers who start a trial with no preference rarely
develop a strong preference subsequently. This does not weaken the author’s argument, though, as
this choice does not suggest that such completely ambivalent researchers are commonplace.
 Cking14
  • Posts: 35
  • Joined: Mar 30, 2015
|
#19879
Hi,

I chose (E) for this question, but the correct answer for this question is (A). I really had trouble with this passage and got the main point question incorrect. What exactly is the author arguing here?

Please help!

Thanks,
Chris
 Steve Stein
PowerScore Staff
  • PowerScore Staff
  • Posts: 1153
  • Joined: Apr 11, 2011
|
#19891
Hi,

That's a good question, about a passage that a lot of students found challenging. In that one, the author discusses an ethical issue involved with comparative medical studies: When there is a traditionally accepted treatment available, doctors and ethicists have generally agreed that participating physicians should have absolutely no opinion as to what treatment is superior. This ambivalent state of mind is referred to as “equipoise.” The author believes that the traditional definition may be too strict. Such a state implicitly requires a perfect balance of evidence for each treatment—a standard which, the author notes, is nearly unattainable. The author's suggestion is that a different standard be adopted. “Clinical equipoise,” says the author, would impose standards that are rigorous but not overly restrictive, especially considering that one reason for comparative trials is to resolve conflicts in expert opinion and in the interpretation of available evidence. In the closing paragraph, the author goes on to say that equipoise is possible because of the conflict in opinion that is an inherent part of such comparative studies.

Question #21 requires you to find the answer choice that would weaken the author’s argument in the last two paragraphs; that is where the author asserts that the notion of clinical equipoise should be adopted, in part because the absence of consensus among experts makes clinical equipoise possible.

Part of the author’s argument is that lack of consensus makes clinical equipoise practicable, because participants can have a preference but recognize that experts disagree. If, as answer choice (A) provides, most comparative clinical trials seek to confirm the preferability of a treatment that is considered the best by a consensus of doctors, the author’s point no longer holds.

Tough question! I hope this is helpful--please let me know whether this is clear--thanks!

~Steve
 Cking14
  • Posts: 35
  • Joined: Mar 30, 2015
|
#19946
Thanks for the explanation! That's a lot of info to juggle and the way you explained it makes sense, but the passage as a whole is just difficult for me! I don't know if I would have ever gotten it write. I am going to re-read the passage and take your notes and apply them to get the full context.

Thanks again!
 emilysnoddon
  • Posts: 64
  • Joined: Apr 22, 2016
|
#26169
I eliminated answer choice A because I felt as though if this answer choice were true it would violate both of the types of equipoise discussed in the passage and therefore be unethical. I wasn't sure how to understand this answer choice, even with the explanation above. I also chose answer choice E because I felt that if it rarely occurred that researches who begin the trial with no preference for either of the treatments later develop a strong preference on the basis of data obtained early in the study then there would be no need for redefining the standard of equipoise that is the purpose of the passage and therefore weaken the argument.

Can someone please explain where my thought process is going wrong?

Thanks!
 Shannon Parker
PowerScore Staff
  • PowerScore Staff
  • Posts: 147
  • Joined: Jun 08, 2016
|
#26286
Hi-

This passage is difficult because it focuses on an area of technical expertise which throws off a lot of students. Try not to worry too much about the subject matter of the passage, beyond what is written or directly implied, and just focus on what the question is asking you to do.

In question 21, you are being directed to figure out exactly what argument the author is making in the third and fourth paragraphs, and then determine which answer choice would most weaken that argument. I see why you found (E) attractive and it does weaken the argument, however it does not address those researches that begin the trial with a preference, and therefore the author's suggestion would still prove useful by addressing those researchers.

Answer choice (A) would in fact, if true, violate both theoretical and clinical equipoise, which would make the author's suggestion completely irrelevant, therefore most weakening his argument, which is why it is the correct answer choice.

I hope this clears it up some.

-Shannon
User avatar
 ashpine17
  • Posts: 321
  • Joined: Apr 06, 2021
|
#90044
I thought E was correct because if true, then perhaps the problems with theoretical equipoise dont' happen as often so we don't need clinical equipoise?
User avatar
 ArizonaRobin
  • Posts: 34
  • Joined: Aug 17, 2019
|
#90948
ashpine17 wrote: Sun Aug 29, 2021 6:39 pm I thought E was correct because if true, then perhaps the problems with theoretical equipoise dont' happen as often so we don't need clinical equipoise?
I did too, but looking at this answer choice in more depth, it can be eliminated because it says that researchers rarely "develop a strong preference" for one option over the other. The author pointed out that the problem with theoretical equipoise is that it can be violated even if a researcher develops a slight preference for one treatment over the other. This answer choice doesn't talk about that scenario, but only addresses developing a strong preference so it doesn't eliminate the need for clinical equipoise.
 Robert Carroll
PowerScore Staff
  • PowerScore Staff
  • Posts: 1787
  • Joined: Dec 06, 2013
|
#91446
ash,

Answer choice (E) doesn't weaken the author's argument because it simply allows theoretical equipoise to be used in those situations. The author isn't against that - the author just thinks that, in some situations, theoretical equipoise will be impossible to meet, so that a looser notion, clinical equipoise, is necessary to fill in the gaps. Answer choice (E) is talking about situations where there are no such gaps, although, note, it's not even saying how often that happens - "researchers who begin with no preference" aren't even quantified, so this could end up being a small percent of researchers, necessitating clinical equipoise to fill in the gaps.

Robert Carroll
 jojapaych
  • Posts: 4
  • Joined: Jul 25, 2020
|
#91490
Hi Robert,

I would actually disagree with this part: "Answer choice (E) doesn't weaken the author's argument because it simply allows theoretical equipoise to be used in those situations."

I would say this answer choice has no effect on either EQU-TH or EQU-CL. For EQU-TH, I think this is so because whether or not a strong preference develops on the part of participants doesn't mean that no preference to a lesser degree develops. And we know that even "small accretions" of evidence tipped in favor of/against a treatment could break the clinical trial under EQU-TH. Moreover, just because preferences don't develop early on don't mean they won't develop at a later stage in the trial. Hence I see (E) not bashing or having any real effect on EQU-TH, either.

My view on why EQU-CL isn't weakened by (E) is that it just doesn't speak on the support offered for EQU-CL. I think that may have been mentioned in a previous comment, but essentially, in the situation (E) presents, we just don't know whether the "lack of consensus" in the clin. comm. that EQU-CL depends on is there or absent, and hence whether the right degree of ethical rigor is there to support using EQU-CL. Since (E) is silent on the argument, I see it once again having no effect whatsoever.

Let me know if I've missed something!

Get the most out of your LSAT Prep Plus subscription.

Analyze and track your performance with our Testing and Analytics Package.